LSAT and Law School Admissions Forum

Get expert LSAT preparation and law school admissions advice from PowerScore Test Preparation.

 Administrator
PowerScore Staff
  • PowerScore Staff
  • Posts: 8917
  • Joined: Feb 02, 2011
|
#25647
Complete Question Explanation
(The complete setup for this game can be found here: lsat/viewtopic.php?t=7800)

The correct answer choice is (D)

This question stem establishes that T is visited fifth. Since the same destination cannot be visited
in two consecutive weeks, it follows that T cannot be visited fourth. Additionally, visiting J
requires the creation of a GJ Block, which would be impossible if J were visited six:
june07_game_3_#13_diagram_1.png
At this point, it would be critical to focus on the most restricted variable in our game – J. J cannot
be visited fourth (first rule), and it also cannot be visited first as a result of the fourth rule.
Therefore, J must be visited either second or third, but not both (which would violate both the
fourth and the fifth rules). In either case, we must ensure that G is visited immediately before J:
june07_game_3_#13_diagram_2.png
At this point, we need to consider the M > G > M sequence. In the first local diagram above, the
sequence can be satisfied if G is also visited fourth (remember – each destination can be visited
more than once):
june07_game_3_#13_diagram_3.png
In the second local diagram, M must be visited first, and again either fourth or sixth. If M is
visited fourth, the sixth destination would be none other than G; likewise, if M is visited sixth, the
fourth destination must be G:
june07_game_3_#13_diagram_4.png
These solutions show that answer choice (D) could be true, as M could be visited fourth. None of
the other answer choices are possible under the condition established by the question stem.
You do not have the required permissions to view the files attached to this post.
 Haleyeastham
  • Posts: 33
  • Joined: Aug 03, 2015
|
#19238
This game refers to the 7 week long voyages. 13 is giving me some trouble. Can you please help explain? Thanks
 jeff.wren
PowerScore Staff
  • PowerScore Staff
  • Posts: 26
  • Joined: Jul 04, 2015
|
#19241
Hi Haley,

Game 3 of the June 2007 LSAT is a basic linear: unbalanced underfunded game where some of the variables will go more than once in order to fill in the 7 spaces. Since some of the variables go more than once, the game involves numerical distributions.

Question 13 is a local could be true question. The first step is to create a miniature diagram with the new information as well as the general information from the main diagram. Create a 1-7 linear diagram and put T in 5 (the new information) and put T in 7 (from rule 2).

HINT: You can check the diagrams above in the first post to see how this looks!

With T in 5, we can add "T" Not Laws under spaces 4 and 6 because of rule 5. We also know that J cannot go in space 6 because that would force G into space 5 (from rule 4). Space 6 is now limited to a G/M dual option.

We still have to satisfy the M>G>M rule and the GJ block. Since J cannot go in space 4 (from rule 1), one way that these remaining variables can fit is:

..... M in 1, GJ in 2 and 3, and then an M/G dual option in space 4.

Answer choice (D) tests the M/G dual option in space 4 (by stating that M can be in 4) and is correct. The other four answers here all present impossible scenarios, and can be ruled out.

Hope this helps,
Jeff
 martinbeslu
  • Posts: 49
  • Joined: Aug 09, 2017
|
#43281
Hi Jeff, I thought G could also go in week 1?
 Jon Denning
PowerScore Staff
  • PowerScore Staff
  • Posts: 904
  • Joined: Apr 11, 2011
|
#43644
Hi Martin - good catch here! You're correct that that isn't the "only" arrangement that could work for this question...as the post up top shows it could also be the case that the GJ block fills spaces 1 and 2, pushing the two Ms further down.

I think Jeff was simply outlining why answer choice (D) works as the only answer that is possible, but I'll change his answer to reflect that as one of two possible placements so that it's more clear!

Thanks for the heads up, and nice work!
 lsatryan
  • Posts: 8
  • Joined: Nov 09, 2020
|
#80864
Hello,

I am having trouble with this question. Why is A not viable as an answer?

I came up with this diagram:

TGMGJMT

It respects the M-G-M rule and the GJ

Please clarify thank you
 Adam Tyson
PowerScore Staff
  • PowerScore Staff
  • Posts: 5153
  • Joined: Apr 14, 2011
|
#80905
Your solution is a valid one in the game overall, lsatryan, but not for this local question, because the question requires us to place T in week 5. We have to work within the local restriction set by the question! Never lose sight of that when doing a local question, and in fact begin your local diagram by complying with that local restriction - place T in the 5th spot and then ask yourself "now what do I do?" At that point you should be thinking about possible placements for the GJ block and for the M-G-M sequence, and you will come up with the two possibilities diagrammed at the top of this thread. J in the 5th spot simply doesn't make that cut here, even though it might be acceptable in answering a different question.

Get the most out of your LSAT Prep Plus subscription.

Analyze and track your performance with our Testing and Analytics Package.